Cardinality of ${emptyset, mathcal{P}(emptyset), {emptyset} } $












8












$begingroup$


This is an old exam question from "Diskrete Mathematik" at ETH Zurich



$mathcal{P}(A)$ denotes the Powerset, which is the set of all subsets of A:



$mathcal{P}(A) := {S|Ssubseteq A }$.



I believe that ${emptyset, mathcal{P}(emptyset), {emptyset} } $ since
$mathcal{P}(emptyset) = {emptyset} $ can be reduced to ${emptyset, {emptyset} } $ and thus the cardinality of said set would be 2. However, the inofficial solution is 3.










share|cite|improve this question











$endgroup$












  • $begingroup$
    Who wrote this unofficial solution? Unofficial solutions have the problem that they might contain typos. I mean, also official solutions can contain typos, but that's another story.
    $endgroup$
    – Asaf Karagila
    Jan 16 '18 at 16:08






  • 2




    $begingroup$
    Can you be absolutely, totally, 100% sure that $mathcal{P}(X)$ denotes the power set of $X$?
    $endgroup$
    – Daniel Fischer
    Jan 16 '18 at 16:09










  • $begingroup$
    @Daniel: Good point, in some old set theory texts, it sometimes appeared as $S(X)$ actually. (I guess "subsets"...)
    $endgroup$
    – Asaf Karagila
    Jan 16 '18 at 16:10












  • $begingroup$
    Your answer, $2$, is correct.
    $endgroup$
    – Andreas Blass
    Jan 16 '18 at 18:31










  • $begingroup$
    @DanielFischer Yes
    $endgroup$
    – Waterhouse
    Jan 26 '18 at 12:30
















8












$begingroup$


This is an old exam question from "Diskrete Mathematik" at ETH Zurich



$mathcal{P}(A)$ denotes the Powerset, which is the set of all subsets of A:



$mathcal{P}(A) := {S|Ssubseteq A }$.



I believe that ${emptyset, mathcal{P}(emptyset), {emptyset} } $ since
$mathcal{P}(emptyset) = {emptyset} $ can be reduced to ${emptyset, {emptyset} } $ and thus the cardinality of said set would be 2. However, the inofficial solution is 3.










share|cite|improve this question











$endgroup$












  • $begingroup$
    Who wrote this unofficial solution? Unofficial solutions have the problem that they might contain typos. I mean, also official solutions can contain typos, but that's another story.
    $endgroup$
    – Asaf Karagila
    Jan 16 '18 at 16:08






  • 2




    $begingroup$
    Can you be absolutely, totally, 100% sure that $mathcal{P}(X)$ denotes the power set of $X$?
    $endgroup$
    – Daniel Fischer
    Jan 16 '18 at 16:09










  • $begingroup$
    @Daniel: Good point, in some old set theory texts, it sometimes appeared as $S(X)$ actually. (I guess "subsets"...)
    $endgroup$
    – Asaf Karagila
    Jan 16 '18 at 16:10












  • $begingroup$
    Your answer, $2$, is correct.
    $endgroup$
    – Andreas Blass
    Jan 16 '18 at 18:31










  • $begingroup$
    @DanielFischer Yes
    $endgroup$
    – Waterhouse
    Jan 26 '18 at 12:30














8












8








8


1



$begingroup$


This is an old exam question from "Diskrete Mathematik" at ETH Zurich



$mathcal{P}(A)$ denotes the Powerset, which is the set of all subsets of A:



$mathcal{P}(A) := {S|Ssubseteq A }$.



I believe that ${emptyset, mathcal{P}(emptyset), {emptyset} } $ since
$mathcal{P}(emptyset) = {emptyset} $ can be reduced to ${emptyset, {emptyset} } $ and thus the cardinality of said set would be 2. However, the inofficial solution is 3.










share|cite|improve this question











$endgroup$




This is an old exam question from "Diskrete Mathematik" at ETH Zurich



$mathcal{P}(A)$ denotes the Powerset, which is the set of all subsets of A:



$mathcal{P}(A) := {S|Ssubseteq A }$.



I believe that ${emptyset, mathcal{P}(emptyset), {emptyset} } $ since
$mathcal{P}(emptyset) = {emptyset} $ can be reduced to ${emptyset, {emptyset} } $ and thus the cardinality of said set would be 2. However, the inofficial solution is 3.







elementary-set-theory






share|cite|improve this question















share|cite|improve this question













share|cite|improve this question




share|cite|improve this question








edited Jan 16 '18 at 16:20







Waterhouse

















asked Jan 16 '18 at 16:04









WaterhouseWaterhouse

1218




1218












  • $begingroup$
    Who wrote this unofficial solution? Unofficial solutions have the problem that they might contain typos. I mean, also official solutions can contain typos, but that's another story.
    $endgroup$
    – Asaf Karagila
    Jan 16 '18 at 16:08






  • 2




    $begingroup$
    Can you be absolutely, totally, 100% sure that $mathcal{P}(X)$ denotes the power set of $X$?
    $endgroup$
    – Daniel Fischer
    Jan 16 '18 at 16:09










  • $begingroup$
    @Daniel: Good point, in some old set theory texts, it sometimes appeared as $S(X)$ actually. (I guess "subsets"...)
    $endgroup$
    – Asaf Karagila
    Jan 16 '18 at 16:10












  • $begingroup$
    Your answer, $2$, is correct.
    $endgroup$
    – Andreas Blass
    Jan 16 '18 at 18:31










  • $begingroup$
    @DanielFischer Yes
    $endgroup$
    – Waterhouse
    Jan 26 '18 at 12:30


















  • $begingroup$
    Who wrote this unofficial solution? Unofficial solutions have the problem that they might contain typos. I mean, also official solutions can contain typos, but that's another story.
    $endgroup$
    – Asaf Karagila
    Jan 16 '18 at 16:08






  • 2




    $begingroup$
    Can you be absolutely, totally, 100% sure that $mathcal{P}(X)$ denotes the power set of $X$?
    $endgroup$
    – Daniel Fischer
    Jan 16 '18 at 16:09










  • $begingroup$
    @Daniel: Good point, in some old set theory texts, it sometimes appeared as $S(X)$ actually. (I guess "subsets"...)
    $endgroup$
    – Asaf Karagila
    Jan 16 '18 at 16:10












  • $begingroup$
    Your answer, $2$, is correct.
    $endgroup$
    – Andreas Blass
    Jan 16 '18 at 18:31










  • $begingroup$
    @DanielFischer Yes
    $endgroup$
    – Waterhouse
    Jan 26 '18 at 12:30
















$begingroup$
Who wrote this unofficial solution? Unofficial solutions have the problem that they might contain typos. I mean, also official solutions can contain typos, but that's another story.
$endgroup$
– Asaf Karagila
Jan 16 '18 at 16:08




$begingroup$
Who wrote this unofficial solution? Unofficial solutions have the problem that they might contain typos. I mean, also official solutions can contain typos, but that's another story.
$endgroup$
– Asaf Karagila
Jan 16 '18 at 16:08




2




2




$begingroup$
Can you be absolutely, totally, 100% sure that $mathcal{P}(X)$ denotes the power set of $X$?
$endgroup$
– Daniel Fischer
Jan 16 '18 at 16:09




$begingroup$
Can you be absolutely, totally, 100% sure that $mathcal{P}(X)$ denotes the power set of $X$?
$endgroup$
– Daniel Fischer
Jan 16 '18 at 16:09












$begingroup$
@Daniel: Good point, in some old set theory texts, it sometimes appeared as $S(X)$ actually. (I guess "subsets"...)
$endgroup$
– Asaf Karagila
Jan 16 '18 at 16:10






$begingroup$
@Daniel: Good point, in some old set theory texts, it sometimes appeared as $S(X)$ actually. (I guess "subsets"...)
$endgroup$
– Asaf Karagila
Jan 16 '18 at 16:10














$begingroup$
Your answer, $2$, is correct.
$endgroup$
– Andreas Blass
Jan 16 '18 at 18:31




$begingroup$
Your answer, $2$, is correct.
$endgroup$
– Andreas Blass
Jan 16 '18 at 18:31












$begingroup$
@DanielFischer Yes
$endgroup$
– Waterhouse
Jan 26 '18 at 12:30




$begingroup$
@DanielFischer Yes
$endgroup$
– Waterhouse
Jan 26 '18 at 12:30










1 Answer
1






active

oldest

votes


















1












$begingroup$

The power set of a finite set with cardinality $x$ has cardinality $2^x$. As you pointed out, ${emptyset, mathcal{P}(emptyset), {emptyset} }$ can be simplified down to ${emptyset, {emptyset} }$, which is equivalent to $mathcal{P}({emptyset})$. Using the property I described initially, we end up with,



$$
2^{|{emptyset}|} = 2^1 = 2.
$$



Therefore, the correct solution is $2$ and not $3$ as the unofficial solution suggested.






share|cite|improve this answer









$endgroup$













    Your Answer





    StackExchange.ifUsing("editor", function () {
    return StackExchange.using("mathjaxEditing", function () {
    StackExchange.MarkdownEditor.creationCallbacks.add(function (editor, postfix) {
    StackExchange.mathjaxEditing.prepareWmdForMathJax(editor, postfix, [["$", "$"], ["\\(","\\)"]]);
    });
    });
    }, "mathjax-editing");

    StackExchange.ready(function() {
    var channelOptions = {
    tags: "".split(" "),
    id: "69"
    };
    initTagRenderer("".split(" "), "".split(" "), channelOptions);

    StackExchange.using("externalEditor", function() {
    // Have to fire editor after snippets, if snippets enabled
    if (StackExchange.settings.snippets.snippetsEnabled) {
    StackExchange.using("snippets", function() {
    createEditor();
    });
    }
    else {
    createEditor();
    }
    });

    function createEditor() {
    StackExchange.prepareEditor({
    heartbeatType: 'answer',
    autoActivateHeartbeat: false,
    convertImagesToLinks: true,
    noModals: true,
    showLowRepImageUploadWarning: true,
    reputationToPostImages: 10,
    bindNavPrevention: true,
    postfix: "",
    imageUploader: {
    brandingHtml: "Powered by u003ca class="icon-imgur-white" href="https://imgur.com/"u003eu003c/au003e",
    contentPolicyHtml: "User contributions licensed under u003ca href="https://creativecommons.org/licenses/by-sa/3.0/"u003ecc by-sa 3.0 with attribution requiredu003c/au003e u003ca href="https://stackoverflow.com/legal/content-policy"u003e(content policy)u003c/au003e",
    allowUrls: true
    },
    noCode: true, onDemand: true,
    discardSelector: ".discard-answer"
    ,immediatelyShowMarkdownHelp:true
    });


    }
    });














    draft saved

    draft discarded


















    StackExchange.ready(
    function () {
    StackExchange.openid.initPostLogin('.new-post-login', 'https%3a%2f%2fmath.stackexchange.com%2fquestions%2f2607924%2fcardinality-of-emptyset-mathcalp-emptyset-emptyset%23new-answer', 'question_page');
    }
    );

    Post as a guest















    Required, but never shown

























    1 Answer
    1






    active

    oldest

    votes








    1 Answer
    1






    active

    oldest

    votes









    active

    oldest

    votes






    active

    oldest

    votes









    1












    $begingroup$

    The power set of a finite set with cardinality $x$ has cardinality $2^x$. As you pointed out, ${emptyset, mathcal{P}(emptyset), {emptyset} }$ can be simplified down to ${emptyset, {emptyset} }$, which is equivalent to $mathcal{P}({emptyset})$. Using the property I described initially, we end up with,



    $$
    2^{|{emptyset}|} = 2^1 = 2.
    $$



    Therefore, the correct solution is $2$ and not $3$ as the unofficial solution suggested.






    share|cite|improve this answer









    $endgroup$


















      1












      $begingroup$

      The power set of a finite set with cardinality $x$ has cardinality $2^x$. As you pointed out, ${emptyset, mathcal{P}(emptyset), {emptyset} }$ can be simplified down to ${emptyset, {emptyset} }$, which is equivalent to $mathcal{P}({emptyset})$. Using the property I described initially, we end up with,



      $$
      2^{|{emptyset}|} = 2^1 = 2.
      $$



      Therefore, the correct solution is $2$ and not $3$ as the unofficial solution suggested.






      share|cite|improve this answer









      $endgroup$
















        1












        1








        1





        $begingroup$

        The power set of a finite set with cardinality $x$ has cardinality $2^x$. As you pointed out, ${emptyset, mathcal{P}(emptyset), {emptyset} }$ can be simplified down to ${emptyset, {emptyset} }$, which is equivalent to $mathcal{P}({emptyset})$. Using the property I described initially, we end up with,



        $$
        2^{|{emptyset}|} = 2^1 = 2.
        $$



        Therefore, the correct solution is $2$ and not $3$ as the unofficial solution suggested.






        share|cite|improve this answer









        $endgroup$



        The power set of a finite set with cardinality $x$ has cardinality $2^x$. As you pointed out, ${emptyset, mathcal{P}(emptyset), {emptyset} }$ can be simplified down to ${emptyset, {emptyset} }$, which is equivalent to $mathcal{P}({emptyset})$. Using the property I described initially, we end up with,



        $$
        2^{|{emptyset}|} = 2^1 = 2.
        $$



        Therefore, the correct solution is $2$ and not $3$ as the unofficial solution suggested.







        share|cite|improve this answer












        share|cite|improve this answer



        share|cite|improve this answer










        answered Dec 25 '18 at 16:39









        EdOverflowEdOverflow

        25119




        25119






























            draft saved

            draft discarded




















































            Thanks for contributing an answer to Mathematics Stack Exchange!


            • Please be sure to answer the question. Provide details and share your research!

            But avoid



            • Asking for help, clarification, or responding to other answers.

            • Making statements based on opinion; back them up with references or personal experience.


            Use MathJax to format equations. MathJax reference.


            To learn more, see our tips on writing great answers.




            draft saved


            draft discarded














            StackExchange.ready(
            function () {
            StackExchange.openid.initPostLogin('.new-post-login', 'https%3a%2f%2fmath.stackexchange.com%2fquestions%2f2607924%2fcardinality-of-emptyset-mathcalp-emptyset-emptyset%23new-answer', 'question_page');
            }
            );

            Post as a guest















            Required, but never shown





















































            Required, but never shown














            Required, but never shown












            Required, but never shown







            Required, but never shown

































            Required, but never shown














            Required, but never shown












            Required, but never shown







            Required, but never shown







            Popular posts from this blog

            Quarter-circle Tiles

            build a pushdown automaton that recognizes the reverse language of a given pushdown automaton?

            Mont Emei